6
$\begingroup$

We say that a sequence $(z_n)$ of real numbers is a modulated Cauchy sequence, whenever there exists a function $\alpha:\mathbb{N} \rightarrow \mathbb{N}$ such that: $$ |z_i-z_j| \le \frac{1}{k} \quad \forall k \ \forall i,j \ge \alpha(k) . $$ Suppose $(b_n)$ is a sequence of real numbers such that for all $n \in \mathbb{N}$, $|b_{n+1}-b_n| \le \left(\frac{1}{2}\right)^n|x|$ for some $x \in \mathbb{R}$.

I want to constructively prove that $(b_n)$ is a modulated Cauchy sequence, without using the axiom of countable choice. I proved this, using the Archimedean property for real numbers; Here is my proof:

By assumption, for all $n \in \mathbb{N}$, $|b_{n+1}-b_n| \le \left(\frac{1}{2}\right)^n|x|$. By Archimedean property, choose a natural number $l$ such that $|x| < l$. Let $i,j \ge 2lk$. put $n=\min\{i,j\}$, $m=\max\{i,j\}$. We have $$ \begin{split} |b_i-b_j| & =|b_m-b_n| \\ & \le |b_m-b_{m-1}|+|b_{m-1}-b_{m-2}|+\dots+|b_{n+1}-b_n| \\ & \le |x|\left(\frac{1}{2}\right)^{m-1}+|x|\left(\frac{1}{2}\right)^{m-2}+\dots+|x|\left(\frac{1}{2}\right)^{n} \\ & =|x|\frac{1+2+2^2+2^3+\dots+2^{m-1-n}}{2^{m-1}} \\ & =|x|\frac{2^{m-n}-1}{2^{m-1}} =|x|\left(2^{1-n}-\frac{1}{2^{m-1}}\right) \\ & \le l \cdot 2^{1-n} \le l \cdot 2^{1-2lk} \le l \cdot \left(\frac{2}{2^{2lk}}\right) \le l \cdot \left(\frac{2}{2lk}\right) \le \frac{1}{k} \text.\\ \end{split} $$ So $\alpha: \mathbb{N} \to \mathbb{N}$ by $\alpha(k)=2lk$ for all $k \in \mathbb{N}$, is a modulus for $(b_n)$.

My question is, is it necessary to use Archimedean property? I wonder what happens if we substitute Archimedean property with weak Archimedean property that is: $$ \forall x \in \mathbb{R} \neg \neg \exists l \in \mathbb{N} \ x<l $$

My motivation for considering weak Archimedean property instead of Archimedean property is that $\mathbb{R}^e$ ("extended real numbers" or "classical real numbers") described by Troelstra and van Dalen in the book "Constructivism in mathematics", is weakly Archimedean but is not Archimedean.

$\endgroup$
9
  • 1
    $\begingroup$ The only reason you're carrying $|x|$ around in your preliminary argument is that $|x|$ might not be invertible, right? The Archimedean property ensures that $|x|$ is invertible iff $x \ne 0$. $\endgroup$ Commented Oct 13, 2024 at 6:56
  • $\begingroup$ Yes, I think if we replace Archimedean property with weakly Archimedean property, we lose the statement that every nonzero real number is invertible. But, I didn't carrying $|x|$ around because of it. I did this, because in my original problem, $|b_{n+1}-b_n| \le (\frac{1}{2})^n |x|$ in which $x$ is an arbitrary nonzero real number. $\endgroup$ Commented Oct 13, 2024 at 8:34
  • 1
    $\begingroup$ Please don't cross post. $\endgroup$ Commented Oct 13, 2024 at 8:59
  • 4
    $\begingroup$ weakly is an adverb and weak is an adjective, whence "weakly Archimedean" but "weak Archimedean property". $\endgroup$ Commented Oct 13, 2024 at 9:27
  • $\begingroup$ @YCor Could one perhaps argue that the intent is "the property (of being) weakly archimedean", rather than "the weak (version of the) archimedean property"? $\endgroup$ Commented Oct 14, 2024 at 18:53

2 Answers 2

1
$\begingroup$

The Archimedean property in its stronger form is in fact necessary. To see this, consider any real number $ x $ (in your favorite system of real numbers $ \mathbb R $). Let $ b _ n = \frac { | x | } { 2 ^ { n - 1 } } $ for all natural numbers $ n $. Then you have $ b _ n - b _ { n + 1 } = \frac { | x | } { 2 ^ n } $, and therefore $ ( b _ n ) $ is asymptotically regular (see Paulo Oliva's comment). Now, note that if $ ( b _ n ) $ is a Cauchy sequence (regardless of having a modulus), then there is a positive integer $ N $ such that $ b _ N - b _ { N + 1 } \le 1 $, or equivalently $ | x | \le 2 ^ N $. Hence, if every asymptotically regular sequence in $ \mathbb R $ is Cauchy, then $ \mathbb R $ satisfies the stronger form of the Archimedean property.

$\endgroup$
2
  • $\begingroup$ Nice answer. It solves the problem. We can't show that $(b_n)$ is a Cauchy sequence (regardless of having a modulus) using weak Archimedean property instead of Archimedean property. Thanks. $\endgroup$ Commented Oct 19, 2024 at 2:52
  • 1
    $\begingroup$ @MohammadTahmasbi You're welcome. Indeed, if you're familiar with models in which the "classical reals" $ \mathbb R ^ { \text e } $ are not isomorphic to the "bounded extended reals" $ \mathbb R ^ { \text {be} } $ (also called McNeil reals), you can find there asymptomatically regular sequences of classical reals that are not convergent (and thus not Cauchy either). $\endgroup$ Commented Oct 19, 2024 at 20:51
1
$\begingroup$

By the double negation translation, when you replace the assumption $\forall x \exists l (x < l)$ by the weaker $\forall x \neg \neg \exists l (x < l)$, then you should still be able to derive the double negation of the conclusion, i.e., $\neg \neg \forall k \forall i, j \geq \alpha(k) (|z_i - z_j| \leq 1/k)$. In intuitionistic logic one always has $\neg \neg \forall a A(a) \to \forall a \neg \neg A(a)$, so you would actually also have $\forall k \forall i, j \geq \alpha(k) \neg \neg (|z_i - z_j| \leq 1/k)$. If, moreover, you dig into the representation of the reals by sequences of rationals, you'll see that the inequality $|z_i - z_j| \leq 1/k$ is also a universal statement ($\Pi^0_1$). So, intuitionistically it would also be acceptable (via "realizability" interpretation) to conclude the original statement $\forall k \forall i, j \geq \alpha(k) (|z_i - z_j| \leq 1/k)$.

$\endgroup$
4
  • $\begingroup$ Thank you. Yes you are right; if we have the function $\alpha$, it will be a modulus for the sequence $(b_n)$ by your argument. But, what is the definition of $\alpha$? In my proof, I have defined $\alpha(k)=l.2k$ for all $k \in \mathbb{N}$. because there was such "$l$". But now, we don't know if there is such $l$. So, how should we define the function $\alpha$? $\endgroup$ Commented Oct 15, 2024 at 12:03
  • 1
    $\begingroup$ Hi Mohammad, quite right, the $\alpha$ depends on $l$. I think the modulus of Cauchyness will indeed need to depend on a bound on $|x|$. If you don't have that bound $l$, then it would not possible to explicitly give the modulus of Cauchyness for the sequence $(b_n)$. By the way, the property you assume about $(b_n)$ is normally called "asymptotic regularity", and with the bound $l$ on $x$ the rate of asymptotic regularity for $(b_n)$ can be given explicitly. $\endgroup$ Commented Oct 16, 2024 at 14:10
  • $\begingroup$ Yes I think you are right. What about Cauchyness? Can we prove that $(b_n)$ is a Cauchy sequence (without modulus) by using weak Archimedean property instead of Archimedean property? $\endgroup$ Commented Oct 17, 2024 at 17:16
  • 1
    $\begingroup$ @MohammadTahmasbi Look at my answer to see that the Archimedean property is in fact necessary for proving Cauchiness. $\endgroup$ Commented Oct 18, 2024 at 15:45

You must log in to answer this question.

Start asking to get answers

Find the answer to your question by asking.

Ask question

Explore related questions

See similar questions with these tags.